LSAT and Law School Admissions Forum

Get expert LSAT preparation and law school admissions advice from PowerScore Test Preparation.

User avatar
 Dave Killoran
PowerScore Staff
  • PowerScore Staff
  • Posts: 5852
  • Joined: Mar 25, 2011
|
#46113
Complete Question Explanation
(The complete setup for this game can be found here: lsat/viewtopic.php?t=17052)

The correct answer choice is (C)

Template #1 eliminates answer choice (A).

Template #2 or #3 can be used to eliminate answer choice (B).

Checking all four templates proves that answer choice (C) is correct.

Template #1 eliminates answer choice (D).

Template #4 (and less resoundingly, template #2) eliminates answer choice (E).
 ashahab1309
  • Posts: 5
  • Joined: Jun 15, 2020
|
#76250
Is there a simpler way to complete this question if we didn't originally draw out the templates?
 Rachael Wilkenfeld
PowerScore Staff
  • PowerScore Staff
  • Posts: 1358
  • Joined: Dec 15, 2011
|
#76288
Hi ashahab1309,

The templates are probably the easiest way to see it. However, if you did not set this game up that way, there's another way to see that answer choice (C) is the one that cannot be true.

The key is in the SS block. You have to have two sales tours next to each other, and no other sales tours. The only way to do that is to have a sales tour on Tuesday (with the other tour on M/W) or a sales tour on Thursday (with the other tour on W/F). That's the only way to have the two sales next to each other. That means, no matter what, you have a sales on Tuesday or Thursday. You also can only have a sales on ONE of Tuesday or Thursday, because the sales tours must be consecutive, and there are only two of them.

That gives you answer choice (C)---you can't have the same tour on Tuesday as Thursday.

Hope that helps!
Rachael

Get the most out of your LSAT Prep Plus subscription.

Analyze and track your performance with our Testing and Analytics Package.